Đến nội dung

Juliel nội dung

Có 1000 mục bởi Juliel (Tìm giới hạn từ 19-04-2020)



Sắp theo                Sắp xếp  

#651206 gõ thử công thức toán

Đã gửi bởi Juliel on 25-08-2016 - 14:20 trong Thử các chức năng của diễn đàn

Đặt $AI=a,IB=b,IC=c,ID=d$ thì ta có $a+b=c+d=1$.

Theo định lí hàm số cos, ta tính được :

$$AC=\sqrt{a^2+c^2-ac},BD=\sqrt{b^2+d^2-bd}$$

Áp dụng BĐT Minkovsky $\sqrt{A^2+B^2}+\sqrt{C^2+D^2}\geq \sqrt{(A+C)^2+(B+D)^2}$, ta có :

$$AC+BD=\sqrt{a^2+c^2-ac}+\sqrt{b^2+d^2-bd}=\sqrt{\left ( a-\frac{c}{2} \right )^2+\left ( \dfrac{\sqrt{3}}{2}c \right )^2}+\sqrt{\left ( b-\frac{d}{2} \right )^2+\left ( \dfrac{\sqrt{3}}{2}d \right )^2}\geq \sqrt{\left ( a+b-\frac{c+d}{2} \right )^2+\left [ \frac{\sqrt{3}}{2}(c+d) \right ]^2}=1$$




#633084 Tổng hợp các bài BĐT trong các đề thi thử THPT Quốc Gia môn Toán năm 2017

Đã gửi bởi Juliel on 14-05-2016 - 17:15 trong Bất đẳng thức và cực trị

Bài 80: Cho $a,b,c$ là các số thực dương thỏa mãn $ab+bc+ac \leq 3$. Tìm GTNN của:

 

$$\frac{12}{4ab+(a+b)(c+3)}+\frac{\sqrt{2(a^2+1)(b^2+1)(c^2+1)}}{(a+1)(b+1)}+\frac{1}{2c^2}$$

Lời giải :

 

Ta có :

$$(a^2+1)(b^2+1)=(a+b)^2+(ab-1)^2$$

$$2(c^2+1)=(c+1)^2+(c-1)^2$$

Áp dụng BĐT Cauchy-Schwarz :

$$\sqrt{2(a^2+1)(b^2+1)(c^2+1)}=\sqrt{\left [ (a+b)^2+(ab-1)^2 \right ].\left [ (c+1)^2+(c-1)^2 \right ]}\geq (a+b)(c+1)+(1-ab)(c-1)=(a+b+c+ab+bc+ca+abc+1)-2(1+abc)=(a+1)(b+1)(c+1)-2(1+abc)$$

Từ đó :

$$\dfrac{\sqrt{2(a^2+1)(b^2+1)(c^2+1)}}{(a+1)(b+1)}=\dfrac{(a+1)(b+1)(c+1)-2(1+abc)}{(a+1)(b+1)}=c+1-\dfrac{2(1+abc)}{(a+1)(b+1)}$$

Theo giả thiết :

$$\dfrac{12}{4ab+(a+b)(c+3)}=\dfrac{12}{3(a+b+ab)+(ab+bc+ca)}\geq \dfrac{12}{3(ab+a+b+1)}=\dfrac{4}{(a+1)(b+1)}$$

Từ đó mà :

$$P\geq \frac{2(1-abc)}{(a+1)(b+1)}+c+1+\dfrac{1}{2c^2}$$

Chú ý rằng :

$$3\geq ab+bc+ca\geq 3\sqrt[3]{abc}\Rightarrow abc\leq 1$$

$$c+\dfrac{1}{2c^2}=\dfrac{c}{2}+\dfrac{c}{2}+\dfrac{1}{2c^2}\geq 3\sqrt[3]{\frac{1}{8}}=\dfrac{3}{2}$$

Như vậy ta được :

$$P\geq \dfrac{5}{2}$$

$$MinP=\dfrac{5}{2}\Leftrightarrow a=b=c=1$$




#631945 $3x^2-x+3=\sqrt{3x+1}+\sqrt{3x+4}$

Đã gửi bởi Juliel on 08-05-2016 - 15:26 trong Phương trình - hệ phương trình - bất phương trình

Giải phương trình :

$$3x^2-x+3=\sqrt{3x+1}+\sqrt{3x+4}$$




#630816 Tổng hợp các bài BĐT trong các đề thi thử THPT Quốc Gia môn Toán năm 2017

Đã gửi bởi Juliel on 02-05-2016 - 17:43 trong Bất đẳng thức và cực trị

Bài 50: Cho x,y là các số dương thỏa mãn điều kiện: $x^2+y^2=xy+1$. Tìm giá trị lớn nhất của biểu thức:

$P=\frac{x^3}{y^3+1}+\frac{y^3}{x^3+1}+\frac{24\sqrt{xy}}{x+y+2}$

(Nguồn: Trích trong tuyển tập các đề thi thử đại học 2015 của thầy Phạm Tuấn Khải)

Lời giải :

 

Gỉa thiết đã cho tương đương :

$$x^3+y^3=x+y$$

Ta có :

$$\dfrac{x^3}{y^3+1}+\dfrac{y^3}{x^3+1}=\dfrac{x^3+y^3+1}{x^3+1}+\dfrac{x^3+y^3+1}{y^3+1}-2=\left ( x^3+y^3+1 \right )\left ( \dfrac{1}{x^3+1}+\dfrac{1}{y^3+1} \right )-2=\left ( x^3+y^3+1 \right ).\frac{x^3+y^3+2}{(x^3+1)(y^3+1)}-2$$

Ta cũng chứng minh được :

$$(1+x^3)(1+y^3)\geq \dfrac{(x+y)^3}{2}$$

Thế nên :

$$\dfrac{x^3}{y^3+1}+\dfrac{y^3}{x^3+1}\leq \dfrac{2(x^3+y^3+1)(x^3+y^3+2)}{(x+y)^3}-2=\frac{2(x+y+1)(x+y+2)}{(x+y)^3}-2$$

Và :

$$\dfrac{24\sqrt{xy}}{x+y+2}\leq \dfrac{12(x+y)}{x+y+2}$$

Vậy nếu ta đặt $x+y=t$ thì :

$$P\leq \dfrac{2(t+1)(t+2)}{t^3}+\dfrac{12t}{t+2}-2=f(t)$$

Ta dễ chứng minh được $t\in \left (0,2 \right ]$. Khảo sát hàm số $f(t)$ trên $\left (0,2 \right ]$. Ta được :

$$MaxP=7\Leftrightarrow t=2\Leftrightarrow x=y=1$$




#630807 Tổng hợp các bài BĐT trong các đề thi thử THPT Quốc Gia môn Toán năm 2017

Đã gửi bởi Juliel on 02-05-2016 - 17:09 trong Bất đẳng thức và cực trị

Bài 51: <HSG 11 Hà Tĩnh 2015-2016>

với a,b,c>0 và $a^2+b^2+c^2+2=a^2b^2c^2$

Chứng minh rằng: $abc(a+b+c)\geq2(ab+bc+ca)$
<Nếu lạc chủ đề hoặc đã có xin mod xoá dùm>

Lời giải :

 

Ta có đẳng  thức :

$$\dfrac{xy}{(x+z)(y+z)}+\dfrac{yz}{(y+x)(z+x)}+\dfrac{zx}{(z+y)(x+y)}+\frac{2xyz}{(x+y)(y+z)(z+x)}=1$$

 

Gỉa thiết đã cho viết dưới dạng :

$$\dfrac{1}{a^2b^2}+\dfrac{1}{b^2c^2}+\dfrac{1}{c^2a^2}+\dfrac{2}{a^2b^2c^2}=1$$

Như vậy ta thấy phải tồn tại các số dương $x,y,z$ thoả :

$$a=\sqrt{\dfrac{x+y}{z}},b=\sqrt{\dfrac{y+z}{x}},c=\sqrt{\dfrac{z+x}{y}}$$

Khi đó ta cần chứng minh :

$$\sqrt{\dfrac{x+y}{2}}+\sqrt{\dfrac{y+z}{x}}+\sqrt{\dfrac{z+x}{y}}\geq 2\left ( \sqrt{\dfrac{x}{y+z}}+\sqrt{\dfrac{y}{z+x}}+\sqrt{\dfrac{z}{x+y}} \right )$$

Điều này là đúng vì :

$$\sqrt{\frac{x}{y+z}}\leq \sqrt{\dfrac{x}{\dfrac{1}{2}\left ( \sqrt{y}+\sqrt{z} \right )^2}}=\frac{\sqrt{2x}}{\sqrt{y}+\sqrt{z}}\leq \dfrac{\sqrt{2x}}{4}\left ( \frac{1}{\sqrt{y}}+\dfrac{1}{\sqrt{z}} \right )$$

Thiết lập các kết quả tương tự rồi cộng vế theo vế, ta thu được điều phải chứng minh.




#630641 Tổng hợp các bài BĐT trong các đề thi thử THPT Quốc Gia môn Toán năm 2017

Đã gửi bởi Juliel on 01-05-2016 - 21:08 trong Bất đẳng thức và cực trị

Bài 46:

Cho x,y,z là các số thực dương thỏa mãn: $x+y+1=z$ . Tìm giá trị nhỏ nhất của biểu thức:
$P=\frac{x}{x+yz}+\frac{y}{y+zx}+\frac{z^2+2}{z+xy}$

Lời giải :

 

Theo BĐT Cauchy-Schwarz :

$$\dfrac{x}{x+yz}+\dfrac{y}{y+zx}\geq \dfrac{(x+y)^2}{x^2+y^2+2xyz}=\dfrac{(x+y)^2}{x^2+y^2+2xy(x+y+1)}=\dfrac{(x+y)^2}{(x+y)^2+2xy(x+y)}=\dfrac{x+y}{x+y+2xy}\geq \dfrac{x+y}{x+y+\dfrac{1}{2}(x+y)^2}=\dfrac{2}{x+y+2}=\dfrac{2}{z+1}$$

Theo AM-GM :

$$\dfrac{z^2+2}{z+xy}\geq \dfrac{4(z^2+2)}{4z+(x+y)^2}=\frac{4(z^2+2)}{4z+(1-z)^2}=\frac{4(z^2+2)}{(z+1)^2}$$

Từ đó :

$$P\geq \dfrac{2}{z+1}+\dfrac{4(z^2+2)}{(z+1)^2}=f(z)$$

Khảo sát hàm số $f(z)$ với $z>0$ ta được :

$$MinP=\dfrac{13}{4}\Leftrightarrow x=y=1,z=3$$




#630627 Tổng hợp các bài BĐT trong các đề thi thử THPT Quốc Gia môn Toán năm 2017

Đã gửi bởi Juliel on 01-05-2016 - 20:48 trong Bất đẳng thức và cực trị

Bài 45: Đề thi thử trường Đại học Hồng Đức

Cho các số thực không âm x,y,z thỏa mãn $x+y+z=3$

Tìm giá trị nhỏ nhất của:

$P= \frac{3}{2}(xyz)^2 +x^{3}+y^{3}+z^{3} -xy-yz-zx +\sqrt{x}+\sqrt{y}+\sqrt{z}$

Lời giải :

 

Ta sẽ chứng minh hai kết quả sau :

 

   Nếu $x,y,z>0$ thoả $x+y+z=3$ thì :

 

$(1)$                     $\sqrt{x}+\sqrt{y}+\sqrt{z}\geq xy+yz+zx$

 

$(2)$                     $x^3+y^3+z^3+\dfrac{3}{2}(xyz)^2\geq \dfrac{9}{2}$

 

Chứng minh (1) :

 

 

Bất đẳng thức tương đương :

$$\sqrt{x}+\sqrt{y}+\sqrt{z}\geq \dfrac{1}{2}\left [ (x+y+z)^2-x^2-y^2-z^2 \right ]\Leftrightarrow x^2+y^2+z^2+2\sqrt{x}+2\sqrt{y}+2\sqrt{z}\geq 9$$

Điều này đúng theo BĐT AM-GM :

$$x^2+\sqrt{x}+\sqrt{x}\geq 3x$$

$$y^2+\sqrt{y}+\sqrt{y}\geq 3x$$

$$z^2+\sqrt{z}+\sqrt{z}\geq 3x$$

 

Chứng minh (2) :

 

 

Đặt $p=x+y+z=3$, $q=xy+yz+zx$ và $r=xyz$. Áp dụng BĐT Schur :

$$r\geq \dfrac{4pq-p^3}{9}=\dfrac{4q-9}{3}$$

Cũng có :

$$x^3+y^3+z^3=3xyz+(x+y+z)\left [ (x+y+z)^2-3(xy+yz+zx) \right ]=3r+3(9-3q)=3r-9q+27$$

Suy ra :

$$x^3+y^3+z^3+\dfrac{3}{2}(xyz)^2=(3r-9q+27)+\dfrac{3}{2}r^2\geq 3.\frac{4q-9}{3}-9q+81+\dfrac{3}{2}.\left ( \dfrac{4q-9}{3} \right )^2=\dfrac{8}{3}q^2-17q+\dfrac{63}{2}$$

Vậy ta chỉ cần chứng minh :

$$\dfrac{8}{3}q^2-17q+\dfrac{63}{2}\geq \dfrac{9}{2}\Leftrightarrow q\leq 3\;\vee q\geq \dfrac{27}{8}$$

Điều này là đúng vì $q=xy+yz+zx\leq \dfrac{1}{3}(x+y+z)^2=3$.

 

Kết luận :

$$MinP=\frac{9}{2}\Leftrightarrow x=y=z=1$$




#630243 Tổng hợp các bài BĐT trong các đề thi thử THPT Quốc Gia môn Toán năm 2017

Đã gửi bởi Juliel on 29-04-2016 - 21:47 trong Bất đẳng thức và cực trị

Bài 43 (Thi thử THPT Quốc gia 2016 THPT Lý Tự Trọng, Nam Định)

Cho $a,b,c$ dương thoả $a+b+c=1$. Tìm GTNN :

$$P=\dfrac{a^2}{(1-a)^2+5bc}+\dfrac{16b^2-27(a+bc)^2}{36(a+c)^2}$$

 

Bài 44. Cho $a,b,c$ dương có tích bằng $1$. Tìm GTNN :

$$P=\dfrac{x^3+1}{\sqrt{x^4+y+z}}+\dfrac{y^3+1}{\sqrt{y^4+z+x}}+\dfrac{z^3+1}{\sqrt{z^4+x+y}}-\dfrac{8(xy+yz+zx)}{xy+yz+zx+1}$$




#630241 Tổng hợp các bài BĐT trong các đề thi thử THPT Quốc Gia môn Toán năm 2017

Đã gửi bởi Juliel on 29-04-2016 - 21:38 trong Bất đẳng thức và cực trị

Bài 42 (THPT Chuyên Lương Văn Tụy-Ninh Bình-2016)

 

Cho $a,b,c\in[1;+\infty )$ thỏa mãn $3(a+b+c)=a^2+b^2+c^2+2ab$.Tìm GTNN của biểu thức:

 

$$P=\frac{a^2}{(a+b)^2+a}+\frac{a}{a+c^2}$$

 

Lời giải :

 

Gỉa thiết đã cho có thể viết dưới dạng :

$$3(a+b+c)=(a+b)^2+c^2$$

Áp dụng BĐT AM-GM :

$$3(a+b+c)=(a+b)^2+c^2\leq \dfrac{1}{2}(a+b+c)^2\Leftrightarrow a+b+c\leq 6\Leftrightarrow b+c\leq 6-a$$

Theo BĐT Cauchy-Schwarz :

$$P\geq \dfrac{\left ( a+\sqrt{a} \right )^2}{a^2+(b+c)^2+2a}=\dfrac{\left ( a+\sqrt{a} \right )^2}{5a+3(b+c)}\geq \dfrac{\left ( a+\sqrt{a} \right )^2}{5a+3(6-a)}=\dfrac{\left ( a+\sqrt{a} \right )^2}{2a+18}=f(a)$$

Dễ dàng thấy $f(a)$ đồng biến trên $\left [ 1,\infty \right )$ nên $P\geq f(a)\geq f(1)=\dfrac{1}{5}$

Kết luận : 

Gía trị nhỏ nhất của $P$ là $\dfrac{1}{5}$, đạt được khi $a=1,b=2,c=3$.




#629218 $3\sqrt{1+2{{x}^{2}}}+2...

Đã gửi bởi Juliel on 23-04-2016 - 23:24 trong Phương trình - hệ phương trình - bất phương trình

Giải phương trình $3\sqrt{1+2{{x}^{2}}}+2\sqrt{40+9{{\left( x-1 \right)}^{2}}}=5\sqrt{11}$ 

Bài này mình đã dùng máy tính (VINACAL 570ES PLUS II) tìm nghiệm nhưng 6 lần nhấn Shift Solve máy cho ra 6 nghiệm: 0.333333368 ; $\frac{1}{3}$ ; 0.333333361 ; 0.333333369 ; 0.333333362; 0.333333378

Bạn nào biết hướng giải làm sao thì chỉ cho mình với.

Phương trình tương đương :

$$\left ( 3\sqrt{1+2x^2}-\dfrac{6x+9}{\sqrt{11}} \right )+\left ( 3\sqrt{40+9(x-1)^2}+\dfrac{6x-46}{\sqrt{11}} \right )=0$$

$$\Leftrightarrow \dfrac{9(1+2x^2)-\dfrac{1}{11}(6x+9)^2}{3\sqrt{1+2x^2}+\dfrac{6x+9}{\sqrt{11}}}+\dfrac{9(40+9(x-1)^2)-\dfrac{1}{11}(6x-46)^2}{3\sqrt{40+9(x-1)^2}+\dfrac{46-6}{\sqrt{11}}}=0$$

$$\Leftrightarrow \frac{18/11.(3x-1)^2}{3\sqrt{1+2x^2}+\dfrac{6x+9}{\sqrt{11}}}+\frac{40/11.(3x-1)^2}{2\sqrt{40+9(x-1)^2}+\dfrac{46-6x}{\sqrt{11}}}=0$$

$$\Leftrightarrow (3x-1)^2\left [ \dfrac{18/11}{3\sqrt{1+2x^2}+\dfrac{6x+9}{\sqrt{11}}}+\dfrac{40/11}{2\sqrt{40+9(x-1)^2}+\dfrac{46-6x}{\sqrt{11}}} \right ]=0$$

 

Dễ dàng chỉ ra với mọi $x$ thực thì :

$$2\sqrt{1+2x^2}+\frac{6x}{\sqrt{11}}> 0$$

$$3\sqrt{40+9(x-1)^2}-\frac{6x}{\sqrt{11}}> 0$$




#629196 Tổng hợp các bài BĐT trong các đề thi thử THPT Quốc Gia môn Toán năm 2017

Đã gửi bởi Juliel on 23-04-2016 - 22:21 trong Bất đẳng thức và cực trị

Bài 40 (Thi thử ĐH 2014 THPT Chuyên Lương Thế Vinh, Đồng Nai)

Cho $a,b,c>0$ thoả $\left ( 3a+2b+c \right )\left ( \dfrac{1}{a}+\dfrac{2}{b}+ \dfrac{3}{c}\right )=30$. Tìm GTLN :

$$P=\frac{b+2c-7\sqrt{72a^2+c^2}}{a}$$




#629151 Tổng hợp các bài BĐT trong các đề thi thử THPT Quốc Gia môn Toán năm 2017

Đã gửi bởi Juliel on 23-04-2016 - 20:39 trong Bất đẳng thức và cực trị

Bài 38 : $Cho x,y,z>0 ; \frac{1}{x}+\frac{1}{y}+\frac{1}{z}=\frac{16}{x+y+z}.MaxP=\frac{(x-y)(y-z)(z-x)}{xyz}$(Thi thử sở Hà tĩnh)

Lời giải : 

 

Gỉa thiết đã cho có thể viết được dưới dạng :

$$\dfrac{x}{y}+\dfrac{y}{x}+\dfrac{y}{z}+\dfrac{z}{x}+\dfrac{z}{x}+\dfrac{x}{z}=13$$

Và : 

$$P=\left ( 1-\dfrac{y}{x} \right )\left ( 1-\dfrac{z}{y} \right )\left ( 1-\dfrac{x}{z} \right )$$

Đặt $a=\dfrac{y}{x},b=\dfrac{z}{y},c=\dfrac{x}{z}$ thì $abc=1$. Gỉa thiết đã cho trở thành $a+b+c+ab+bc+ca=13$.

Và biểu thức trở thành :

$$P=(1-a)(1-b)(1-c)=(1-abc)+(ab+bc+ca-a-b-c)=13-2(a+b+c)$$

Sử dụng hai giả thiết :

$$(a+b)+c(a+b)+\dfrac{1}{c}+c=13\Leftrightarrow (a+b)(c+1)=13-\dfrac{1}{c}-c\Leftrightarrow a+b=\dfrac{13c-c^2-1}{c(c+1)}$$

Thay vào $P$ :

$$P=13-2\left ( \frac{13c-c^2-1}{c(c+1)}+c \right )=\dfrac{-2c^3+13c^2-13c+2}{c(c+1)}=f(c),c>0$$

$$f'(c)=\dfrac{-2(c^2-3c+1)(c^2+5c+1)}{c^2(c+1)^2}$$

Từ đó dễ dàng thấy :

$$f_{max}=\sqrt{5}\Leftrightarrow c=\dfrac{3+\sqrt{5}}{2}$$

Từ đó :

Gía trị lớn nhất của $P$ là $\sqrt{5}$, đạt được khi chẳng hạn $a=c=\dfrac{3+\sqrt{5}}{2},b=\dfrac{1}{2}\left ( 7-3\sqrt{5} \right )$ tức $y=\frac{3+\sqrt{5}}{2}.x=\dfrac{7+3\sqrt{5}}{2}.z$




#629136 Tổng hợp các bài BĐT trong các đề thi thử THPT Quốc Gia môn Toán năm 2017

Đã gửi bởi Juliel on 23-04-2016 - 19:53 trong Bất đẳng thức và cực trị

Bài 36: Trích đề thi thử đại học lần 2 trường THPT Đoàn Thượng

Cho $a,b,c>0$ thỏa $a^{2}+b^{2}+c^{2}=3$

TÌm giá trị lớn nhất của:

$A=\frac{ab}{3+c^{2}}+\frac{bc}{3+a^{2}}-\frac{a^{3}b^{3}+b^{3}c^{3}}{24a^{3}c^{3}}$

Lời giải :

 

Ta có :

$$\dfrac{ab}{3+c^2}=\dfrac{ab}{(c^2+a^2)+(c^2+b^2)}\leq \dfrac{1}{4}.\dfrac{(a+b)^2}{(c^2+a^2)+(c^2+b^2)}\leq \dfrac{1}{4}\left ( \dfrac{a^2}{c^2+a^2}+\dfrac{b^2}{c^2+b^2} \right )\leq \dfrac{1}{4}\left ( \dfrac{a^2}{c^2+a^2}+\dfrac{b}{2c} \right )$$

Hoàn toàn tương tự :

$$\dfrac{bc}{3+a^2}\leq \dfrac{1}{4}\left ( \dfrac{c^2}{c^2+a^2}+\dfrac{b}{2a} \right )$$

Suy ra :

$$\dfrac{ab}{3+c^2}+\dfrac{bc}{3+a^2}\leq \dfrac{1}{4}\left ( 1+\dfrac{1}{2}\left ( \dfrac{b}{a}+\dfrac{b}{c} \right ) \right )=\dfrac{1}{4}+\dfrac{1}{8}\left ( \dfrac{b}{a}+\dfrac{b}{c} \right )$$

Và :

$$24A\leq 6+3\left ( \dfrac{b}{a}+\dfrac{b}{c} \right )-\left ( \frac{b^{3}}{a^3}+\dfrac{c^3}{a^3} \right )$$

Dễ dàng chứng minh được :

$$3x-x^3\leq 2\;\;\;\;(x>0)$$

Suy ra :

$$24A\leq 6+\left ( \dfrac{3b}{a}-\dfrac{b^3}{a^3} \right )+\left ( \dfrac{3b}{c} -\dfrac{b^3}{c^3}\right )\leq 6+2+2=10\Leftrightarrow A\leq \dfrac{5}{12}$$

Kết luận :

$$A_{max}=\dfrac{5}{12}\Leftrightarrow a=b=c=1$$




#627815 Tìm GTNN của $P=\frac{a^{2}+b^{2}+c^{...

Đã gửi bởi Juliel on 17-04-2016 - 20:34 trong Bất đẳng thức và cực trị

Chỗ này là sao bạn ơi, một phát ra mà không hiểu lắm

3(a+b+c)2=12+5(ab+bc+ca)12

$a,b,c$ không âm nên $ab+bc+ca \geq 0$




#627813 Tổng hợp các bài BĐT trong các đề thi thử THPT Quốc Gia môn Toán năm 2017

Đã gửi bởi Juliel on 17-04-2016 - 20:29 trong Bất đẳng thức và cực trị

Bài 32 (Thi thử THPT Quốc gia 2016 THPT Chuyên Lương Thế Vinh, Đồng Nai)

Cho $a,b,c$ dương thoả $\dfrac{1}{a}+\dfrac{1}{b}+\dfrac{1}{c}=3$. Tìm GTNN :

$$P=(a+1)(b+1)(c+1)+\dfrac{4}{\sqrt{a^2+b^2+c^2+1}}$$




#627585 Tìm GTNN của $P=\frac{a^{2}+b^{2}+c^{...

Đã gửi bởi Juliel on 16-04-2016 - 20:45 trong Bất đẳng thức và cực trị

hay quá bạn ơi, còn giá trị bé nhất?

Gía trị nhỏ nhất làm như sau.

Ta sẽ chứng minh $a+b+c \geq 2$. 

Từ giả thiết :

$$3(a+b+c)^2=12+5(ab+bc+ca)\geq 12$$

Suy ra $a+b+c \geq 2$. 

Tới đây bạn đưa về xét hàm theo ẩn $t=a+b+c$ là được. $P$ đưa được về hàm đồng biến theo ẩn $t$.




#627575 Tổng hợp các bài BĐT trong các đề thi thử THPT Quốc Gia môn Toán năm 2017

Đã gửi bởi Juliel on 16-04-2016 - 20:21 trong Bất đẳng thức và cực trị

Bài 30: (THPT chuyên Thái Bình- lần 3)

Cho ba số thực dương $x,y,z$ thỏa mãn $x+2y+3z=4$. Tìm giá trị lớn nhất của biểu thức:

$P=x^{2}\left ( 5-6x \right )+4y^{2}\left ( 5-12y \right )+z^{2}\left ( 45-162z \right )$

Lời giải :

 

Đặt $a=x,b=2y,c=3z$ thì $a+b+c=4$. Khi đó :

$$P=a^2(5-6a)+b^2(5-6b)+c^2(5-6c)=5a^2+5b^2+5c^2-6a^3-6b^3-6c^3$$

Bằng phương pháp tiếp tuyến, chỉ ra được :

$$5a^2-6a^3\leq \dfrac{-56}{3}a+\dfrac{176}{9}$$

Suy ra :

$$P\leq \dfrac{-56}{3}(a+b+c)+\dfrac{176}{3}=-16$$

$$MaxP=-16\Leftrightarrow x=\dfrac{4}{3},y=\dfrac{2}{3},z=\dfrac{4}{9}$$




#627569 Tổng hợp các bài BĐT trong các đề thi thử THPT Quốc Gia môn Toán năm 2017

Đã gửi bởi Juliel on 16-04-2016 - 20:13 trong Bất đẳng thức và cực trị

 

Bài 27 (Đề thi thử môn Toán năm 2016 trường THPT ĐăkMil, ĐăkNông)

 

Cho $a,b,c$ là các số thực không âm thỏa mãn:$ab+bc+ac=1$.Tìm Min biểu thức:

 

$$P=\sqrt{\frac{a}{16(b+c)(a^2+bc)}}+\sqrt{\frac{b}{16(a+c)(b^2+ac)}}+\frac{a^2+1}{4}\left ( \frac{1}{a}+\frac{c}{ab} \right )$$

Lời giải :

 

Theo AM-GM :

$$\sqrt{\dfrac{a}{(b+c)(a^2+bc)}}=\dfrac{a}{\sqrt{(ab+ac)(a^2+bc)}}\geq \dfrac{a}{\dfrac{1}{2}(ab+ac+a^2+bc)}=\dfrac{2a}{(a+b)(a+c)}$$

Tương tự :

$$\sqrt{\dfrac{b}{(a+c)(b^2+ac)}}\geq \dfrac{2b}{(b+c)(b+a)}$$

Suy ra :

$$4P\geq \dfrac{2a}{(a+b)(a+c)}+\dfrac{2b}{(b+c)(b+a)}+\dfrac{(a^2+1)(b+c)}{ab}=\dfrac{2a(b+c)+2b(c+a)}{(a+b)(b+c)(c+a)}+\dfrac{(a^2+ab+bc+ca)(b+c)}{ab}$$

$$=\dfrac{2(1+ab)}{(a+b)(b+c)(c+a)}+\dfrac{(a+b)(b+c)(c+a)}{ab}\geq 2\sqrt{\dfrac{2(1+ab)}{ab}}=2\sqrt{\dfrac{4(1+ab)}{2ab}}\geq 2\sqrt{\dfrac{4(ab+1)}{1+ab}}=4$$

(Chú ý rằng $ab\leq ab+bc+ca=1$)

Từ đó ta có $MinP=1\Leftrightarrow a=b=1,c=0$




#621286 Tổng hợp các bài BĐT trong các đề thi thử THPT Quốc Gia môn Toán năm 2017

Đã gửi bởi Juliel on 19-03-2016 - 22:00 trong Bất đẳng thức và cực trị

$\boxed{9}$ (Đề thi thử ĐH môn Toán của SG&ĐT Vĩnh Phúc lần 2 năm 2016)

 

Cho $a,b,c$ dương thoả mãn điều kiện $a^2+b^2+c^2\leq 1$.Tìm giá trị nhỏ nhất của biểu thức: 

 

$$P=\frac{2}{3}\left ( \frac{a}{b^2+c^2}+\frac{b}{c^2+a^2}+\frac{c}{a^2+b^2} \right )-\sqrt{(ab+bc+ac)^3}-2\sqrt{3}\sqrt[3]{abc}$$

Lời giải :

Ta có :

$$\dfrac{a}{b^2+c^2}\geq \dfrac{a}{1-a^2}=\dfrac{a^2}{a(1-a)(1+a)}$$

Lại có :

$$a(1-a)(1+a)=\frac{1}{(2-\sqrt{3})(\sqrt{3}-1)}.(1-x).\left [ (2-\sqrt{3})+(2-\sqrt{3})x \right ].\left ( \sqrt{3}-1 \right )x\leq \frac{1}{27(2-\sqrt{3})(\sqrt{3}-1)}.\left [ (1-x)+(2-\sqrt3)+(2-\sqrt3)x+(\sqrt3-1)x \right ]^3=\dfrac{2\sqrt{3}}{9}$$

Suy ra :

$$\dfrac{a}{b^2+c^2}\geq \frac{a^2}{a(1-a)(1+a)}\geq \dfrac{3\sqrt{3}}{2}a^2$$

Hoàn toàn tương tự với hai phân thức còn lại, ta suy ra :

$$\dfrac{a}{b^2+c^2}+\dfrac{b}{c^2+a^2}+\dfrac{c}{a^2+b^2}\geq \frac{3\sqrt{3}}{2}(a^2+b^2+c^2)$$

Và cũng có :

$$ab+bc+ca\leq a^2+b^2+c^2\Rightarrow -\sqrt{(ab+bc+ca)^3}\geq -\sqrt{(a^2+b^2+c^2)^3}$$

$$a^2+b^2+c^2\geq 3\sqrt[3]{a^2b^2c^2}\Rightarrow -2\sqrt{3}\sqrt[3]{abc}\geq -2\sqrt{a^2+b^2+c^2}$$

Do vậy nếu đặt $t=\sqrt{x^2+y^2+z^2}\in \left ( 0,1 \right ]$ thì :

$$P\geq \sqrt{3}t^2-t^3-2t=f(t)$$

$$f'(t)=2\sqrt{3}t-3t^2-2<0,\;\forall t\in \left ( 0,1 \right )$$

Vậy $f$ nghịch biến trên $\left (0,1 \right ]$. Từ đó :

$$f(t)\geq f(1)=-3+\sqrt{3}$$

Ta có :

 

$Min P=-3+\sqrt{3}$, đạt được khi $a=b=c=\dfrac{1}{\sqrt{3}}$




#614024 $P=\dfrac{2(xy+yz+zx)}{xyz+2(2x+y+z)}+\dfr...

Đã gửi bởi Juliel on 10-02-2016 - 20:07 trong Bất đẳng thức và cực trị

Cho các số thực $x,y,z$ thoả mãn $x,y,z\in \left [ 1,2 \right ]$. Tìm giá trị lớn nhất của biểu thức :

$$P=\dfrac{2(xy+yz+zx)}{xyz+2(2x+y+z)}+\dfrac{8}{2x(y+z)+yz+4}-\dfrac{y+z+4}{\sqrt{yz}+1}$$




#613002 Tìm số tam giác vuông có 3 đỉnh là đỉnh của đa giác

Đã gửi bởi Juliel on 05-02-2016 - 12:20 trong Tổ hợp - Xác suất và thống kê - Số phức

Xét đa giác đều $12$ đỉnh. Hãy tìm :

a) Số các tam giác vuông có 3 đỉnh là đỉnh của đa giác.

b) Số các tam giác không đều có 3 cạnh là đường chéo của đa giác.




#613001 Trong không gian $Oxyz$ cho mặt phẳng $(P):x-2y+2z+3=0$

Đã gửi bởi Juliel on 05-02-2016 - 12:18 trong Phương pháp tọa độ trong không gian

Trong không gian $Oxyz$ cho mặt phẳng $(P):x-2y+2z+3=0$ và đường thẳng $(d)$ : $\dfrac{x-3}{2}=\dfrac{y+2}{-1}=\dfrac{z+1}{2}$. Viết phương trình đường thẳng $(\Delta)$ nằm trong $(P)$, cắt $(d)$ và cách gốc toạ độ $O$ một khoảng bằng $\sqrt{10}$.




#610821 $\left\{\begin{matrix}y^{3}+y+4=...

Đã gửi bởi Juliel on 24-01-2016 - 20:57 trong Phương trình - Hệ phương trình - Bất phương trình

 

2, $\left\{\begin{matrix}x^{3}-3y^{3}-3x^{2}y+xy^{2}+x=3y & \\ 3x^{3}+36y^{2}-1=x\sqrt[3]{27y^{3}+\frac{2x+1}{x}} & \end{matrix}\right.$

Lời giải :

Điều kiện $x\neq 0$

Phương trình đầu của hệ có thể viết dưới dạng :

$$\left ( x^3-3x^2y \right )+(xy^2-3y^3)+(x-3y)=0\Leftrightarrow (x-3y)(x^2+y^2+1)=0\Leftrightarrow x=3y$$

Thay vào phương trình sau :

$$3x^3+4x^2-1=x\sqrt[3]{x^3+\dfrac{1}{x}+2}\Leftrightarrow 3x^2+4x-\dfrac{1}{x}=\sqrt[3]{x^3+\dfrac{1}{x}+2}\Leftrightarrow \left ( x+1 \right )^3+(x+1)=x^3+\dfrac{1}{x}+2+\sqrt[3]{x^3+\dfrac{1}{x}+2}$$

Bằng cách xét hàm số $f(t)=t^3+t$ với $t\in \mathbb{R}$. Dễ thấy $f$ đồng biến trên $\mathbb{R}$. Do đó :

$$f(x+1)=d\left ( \sqrt[3]{x^3+\dfrac{1}{x}+2} \right )\Leftrightarrow x+1= \sqrt[3]{x^3+\dfrac{1}{x}+2} \Leftrightarrow 3x^3+3x^2-x-1=0\Leftrightarrow x\in \left \{ -1,\dfrac{1}{\sqrt{3}},\dfrac{-1}{\sqrt{3}} \right \}$$

Nghiệm của hệ là :

$$\left ( x,y \right )=\left ( -1,\dfrac{-1}{3} \right ),\left ( \dfrac{1}{\sqrt{3}},\dfrac{1}{3\sqrt{3}} \right ),\left ( \dfrac{-1}{\sqrt{3}},\dfrac{-1}{3\sqrt{3}} \right )$$




#606383 Viết phương trình đường tròn $(C)$ đi qua $A,B$ và cắt...

Đã gửi bởi Juliel on 31-12-2015 - 22:35 trong Phương pháp tọa độ trong mặt phẳng

Trong mặt phẳng toạ độ $Oxy$, cho $A(1,2),B(3,4)$ và đường thẳng $(d) : y-3=0$. Viết phương trình đường tròn $(C)$ đi qua $A,B$ và cắt $(d)$ tại hai điểm $M,N$ thoả mãn $\angle MAN=60^0$




#605527 Max: $P=7(x+2y)-4\sqrt{x^2+2xy+8y^2}$

Đã gửi bởi Juliel on 27-12-2015 - 16:33 trong Bất đẳng thức và cực trị

Cho x,y là các số thực dương thỏa mãn $x^2+2y^2=\frac{8}{3}$ . Tìm giá trị lớn nhất của

$P=7(x+2y)-4\sqrt{x^2+2xy+8y^2}$

Lời giải :

Ta có :

$$4\sqrt{x^2+2xy+8y^2}=\sqrt{16x^2+32xy+128y^2}=\sqrt{7(x-2y)^2+(3x+10y)^2}\geq 3x+10y$$

Suy ra :

$$P\leq 7(x+2y)-(3x+10y)=4(x+y)\leq 4.\sqrt{(1+\dfrac{1}{2})(x^2+2y^2)}=8$$

$$P_{max}=8\Leftrightarrow x=\frac{4}{3},y=\dfrac{2}{3}$$